%I #9 Oct 21 2019 19:12:43
%S 2,4,6,8,9,10,11,12,14,16,17,18,19,20,21,22,24,25,26,27,28,30,32,33,
%T 34,35,36,38,39,40,41,42,43,44,45,46,48,49,50,51,52,53,54,56,57,58,59,
%U 60,62,64,65,66,67,68,69,70,71,72,73,74,75,76,77,78,79,80
%N Complement of A192110.
%C Note that, because A192110 assumes i <= 40, it is incorrect to say that the present sequence consists of "the positive integers that cannot be expressed as 2^m-3^n where m and n are integers".
%C This sequence is included because one way to remove the assumption i <= 40 from A192110 (and the fifty other unproved sequences of the same type) would be to show that the complements are correct, using the method used to prove the correctness of A173671.
%H Math Overflow, <a href="https://mathoverflow.net/questions/29926/3n-2m-pm-41-is-not-possible-how-to-prove-it/29956#29956">3^n - 2^m = +-41 is not possible. How to prove it?</a>, Several contributors, Jun 29 2010.
%Y Complement of A192110.
%Y Cf. A192111, A173671.
%K nonn
%O 1,1
%A _N. J. A. Sloane_, Oct 12 2019
%E Edited by _N. J. A. Sloane_, Oct 21 2019